Z Transforms
Z Transforms
Z-Transforms.
4.1      Definitions
Unilateral Z-transform.
                                                                     T
   Let {f (n)} = f (0), f (1), f (2), · · · be any sequence defined for n = 0, 1, 2, · · · .Then the
                                                                IS
Z-transform of the given sequence { f(n) }is defined as
                                                   ∞
                                                   X        M
                                                         f (n)z −n
                                                   SR
                                      Z[f (n)] =
                                                   n=0
   The right hand side of the above equation is a function of z and hence it is denoted by
Z[f (n)] = F (z).
                           ur
Bilateral Z-transform
                         .S
                      r.E
   Let {f (n)}be a sequence defined for n = 0, ±1, ±2, · · · .Then the Z-transform the given
sequence { f(n) }is defined as
                                            ∞
                  D
                                            X
                                 Z[f (n)] =     f (n)z −n
                                                n=−∞
Note
1. Throughout this chapter we consider only the unilateral Z-transform.
   Basic Formulae:
  1. (1 − z)−1 = 1 + z + z 2 + z 3 + ...
  2. (1 − z)−2 = 1 + 2z + 3z 2 + 4z 3 + ...
                          z2 z3 z4
  3. − log(1 − z) = z +     +   +   + ...
                          2   3   4
                z   z2 z3 z4
  4. ez = 1 +     +   +    +    + ...
                1! 2!   3!   4!
                                                   1
2
                                                                            T
                                          = aF (z) + bG(z)
                                                                       IS
1. Z[1] =
             z                                               M
                                                    SR
            z−1
Proof.
                                              h,
                                   ∞                 ∞  n
                                   X
                                              −n
                                                     X    1
                     Z[f (n)] =     f (n)z =      f (n)
                              es
                                n=0           n=0
                                                          z
                                 ∞  n               2  3
                                       1        1       1        1
                            ur
                                X
                        Z[1] =             =1+ +             +     + ···
                                       z        z       z        z
                          .S
                                n=0
                                        −1         −1
                                       1       z−1             z
                              =   1−        =               =
                       r.E
                                       z          z           z−1
                   D
               z
2. Z[n] =            , |z| > 1
            (z − 1)2
Proof.
                         ∞              ∞     n
                         X
                                  −n
                                        X      1
         Z[f (n)] =     f (n)z =       f (n)
                    n=0            n=0
                                               z
                     ∞       n                  2       3
                    X       1              1        1           1
            Z[n] =      n        =0+1          +2         +3       + ···
                    n=0
                            z              z        z           z
                              −2              −2
                                                      1 z2
                                     
                    1       1       1 z−1                             z
                  =     1−        =                =           2 =          , |z| > 1
                    z       z       z     z           z (z − 1)    (z − 1)2
                                                                                      3
               z
3. Z[an ] =       if |z| > |a|
              z−a
Proof.
                          ∞                    ∞    n
                          X
                                        −n
                                               X    1
              Z[f (n)] =     f (n)z =        f (n)
                         n=0             n=0
                                                    z
                          ∞      n X    ∞  
                               n 1              a n
                         X                                a   a 2  a 3
                    n
               Z(a ) =       a         =            =1+        +        +    + ···
                         n=0
                                   z     n=0
                                                z          z       z      z
                                               −1
                               a −1     z−a           z
                       = 1−           =              =       , |z| > |a|
                                z          z           z−a
                 z
4.Z[(−a)n ] =       if |z| > |a|
                z+a
Proof.
                        ∞                    ∞ n
                        X
                                   −n
                                             X  1
          Z[f (n)] =     f (n)z =        f (n)
                                                                         T
                     n=0             n=0
                                                z
                                                                     IS
                      ∞          n X    ∞      n
                n
                     X         n 1             −a          a   a 2  a 3
         Z[(−a) ] =      (−a)         =              =1−        +     −       + ···
                     n=0
                                  z      n=0
                                                z            z   M  z    z
                                                         SR
                                           −1
                           a −1     z+a            z
                   = 1+           =             =      , if |z| > |a|
                            z          z           z+a
                                                 h,
                  
      1           z
5. Z     = log         , |z| > 1
                              es
      n          z−1
                            ur
Proof.
                                         ∞                     ∞
                          .S
                                                                         n
                                         X
                                                        −n
                                                               X         1
                           Z[f (n)] =          f (n)z        =     f (n)
                                                                         z
                       r.E
n=0 n=0
                                 1
                   D
                                                                  T
                                                  z                 z
                                                             IS
                                                 −1                
                                             z−1                  z
                               =    z log              = z log          , |z| > 1
                                               z
                                                        M        z−1
                                                  SR
                    
      1     1       z
7. Z       = log         , |z| > 1
     n−1    z      z−1
                                            h,
Proof.
                              es
                           ∞                ∞           n
                           X
                                       −n
                                            X            1
              Z[f (n)] =       f (n)z =          f (n)
                            ur
                           n=0               n=0
                                                         z
                          .S
                          ∞                n          2      3         4
              1            X        1        1        1 1         1 1         1 1
           Z           =                           =            +          +         + ···
             n−1                  n −  1     z        1    z      2    z      3  z
                       r.E
                           n=2
                             "           2          3          #
                           1     1       1 1         1 1
                       =              +           +            + ···
                 D
                           z     z       2 z         3 z
                                                              
                             1             1                z−1
                       =   − log 1 −           = − log
                             z             z                  z
                                         −1                   
                           1        z−1            1          z
                       =     log               = log                , |z| > 1
                           z          z            z        z−1
       
     1       1
8. Z      = ez
     n!
Proof.
                       ∞              ∞    n
                       X
                               −n
                                      X    1
         Z[f (n)] =     f (n)z =     f (n)
                    n=0          n=0
                                           z
                   ∞      n                 2      3      4
              1     X    1 1           1 1     1 1      1 1      1 1
         Z        =             =1+          +        +        +        + ···
             n!     n=0
                        n! z          1! z     2! z     3! z     4! z
                                                                                            5
                         2  3  4
                       1    1    1    1
                       z    z    z    z             1
                 = 1+     +    +    +    + · · · = ez
                       1!   2!   3!   4!
             
        1         h 1  i
9. Z            =z e −1
                    z
     (n + 1)!
Proof.
                              ∞              ∞           n
                              X
                                       −n
                                             X           1
              Z[f (n)] =           f (n)z =        f (n)
                              n=0             n=0
                                                         z
                             ∞               n
              1               X        1     1
         Z                =
           (n + 1)!           n=0
                                   (n + 1)! z
                                                 2        3
                               1      1 1       1 1         1 1
                          =      +           +           +         + ···
                              1! 2! z          3! z        4! z
                                                                    T
                                "             2         3       4        #
                                  1 1        1 1         1 1       1 1
                          =   z            +           +         +         + ···
                                                              IS
                                  1! z       2! z        3! z      4! z
                          =
                                "
                              z 1+
                                          
                                       1 1
                                               +
                                                     2
                                                  1 1    M +
                                                                3
                                                             1 1
                                                                    +
                                                                          4
                                                                       1 1
                                                                               + ··· − 1
                                                                                        #
                                                  SR
                                       1! z       2! z       3! z      4! z
                                h 1      i
                          =   z ez − 1
                                            h,
                               es
                                                   z
                                w.k.t Z[an ] =
                                                  z−a
                           .S
                                                                   z
                                      Z [ean ] = Z [(ea )n ] =
                        r.E
                                                                 z − ea
                  D
                          z
          w.k.t Z[an ] =
                         z−a                     n 
                 −n      −n     −1 n       1         z        az
               Z a     = Z a     =Z a        =Z         =     1
                                                                =
                                                  a       z− a     az − 1
13. Evaluate Z [n + 3]
                                                                             
                                                           z               z
           Z [n + 3] = Z [n] + Z [3] = Z [n] + 3Z [1] =            +3
                                                        (z − 1)2        (z − 1)
6
                                  let a = eiθ
                                      an = einθ = cos nθ + i sin nθ
                                               z
                           w.k.t Z[an ] =
                                            z−a                        
                                   inθ        iθ n            z
                               Z e        = Z e           =
                                                                z − eiθ
                                                                                                
                                                          z                            z
                                          =                              =
                                              z − (cos θ + i sin θ)          (z − cos θ) − i sin θ
                                                                                                
                                                          z                 (z − cos θ) + i sin θ
                                          =                              ×
                                              (z − cos θ) − i sin θ         (z − cos θ) + i sin θ
                                            z(z − cos θ) + iz sin θ
                                          =
                                                                    T
                                              (z − cos θ)2 + sin2 θ
                                                               IS
                                            z(z − cos θ) + iz sin θ        z(z − cos θ) + iz sin θ
                 Z [(cos nθ + i sin nθ)] =                  2            =
                                                                     2        z 2 − 2z cos θ + 1
                                                           M
                                              (z − cos θ) + sin θ
                                                z(z − cos θ)                z sin θ
                                                     SR
                Z [cos nθ] + iZ [sin nθ] = 2                       +i 2
                                            z − 2z cos θ + 1           z − 2z cos θ + 1
    equating real and imaginary parts
                                               (z 2 − z cos θ)
                                             h,
                              Z [cos nθ] = 2
                                            z − 2z cos θ + 1
                                es
                                                    z sin θ
                              Z [sin nθ] = 2
                                            z − 2z cos θ + 1
                              ur
                            .S
                       z 2 − z cosh θ
15.Z [cosh nθ] =
                         r.E
                   z 2 − 2z cosh θ + 1
Proof. Simply
                   D
                                                     (z 2 − z cos θ)
                                     Z [cos nθ] =
                                                  z 2 − 2z cos θ + 1
                                                     (z 2 − z cosh θ)
                                ∴   Z [cosh nθ] = 2
                                                  z − 2z cosh θ + 1
                          z sinh θ
16.Z [sinh nθ] =
                   z2   − 2z cosh θ + 1
Proof. Simply
                                                      z sin θ
                                      Z [sin nθ] =
                                                     z2
                                                    − 2z cos θ + 1
                                                      z sinh θ
                                ∴   Z [sinh nθ] = 2
                                                  z − 2z cosh θ + 1
                                                                          7
                   h nπ i    z2
17. Prove that Z cos      = 2
                      2    z +1
Solution. We know that
                                                    z 2 − z cos θ
                                 Z [cos nθ] =
                                                z 2 − 2z cos θ + 1
                            z 2 − z cos π2
                                           
               h   nπ i                            z 2 − z(0)      z2
             Z cos      = 2                  =                 =
                         z − 2z cos π2 + 1
                                        
                    2                          z 2 − 2z(0) + 1   z2 + 1
                   h nπ i     z
18. Prove that Z sin      = 2
                      2    z +1
Solution. We know that
                                                       z sin θ
                                 Z [sin nθ] =                       .
                                                z2   − 2z cos θ + 1
                                                                     T
Hence
                                                               IS
                             z sin π2
                                      
                   nπ i                        z(1)          z
               h
             Z sin
                    2
                        = 2           π
                                        
                         z − 2z cos 2 + 1
                                          = 2
                                           z − 2z(0) + 1   M
                                                         = 2
                                                          z +1
                                                 SR
          h     nπ i
19. Find Z sin2
                                          h,
                 2
                              es
                       w.k.tsin2 θ =
                                        2
                          .S
                                         2 nπ     1 h           nπ i
                                    h         i
                                   Z sin        =   Z 1 − cos 2
                                            2     2              2
                       r.E
                                                  1
                                                =   [Z[1] − Z [cos nπ]]
                                                  2
               D
                                                  1
                                                =   [Z[1] − Z [(−1)n ]]
                                                  2                
                                                  1      z      z
                                                =            −
                                                  2 z−1 z+1
                                                          
                                                       z
                                                =
                                                    z2 − 1
8
                                                                  T
20. Find the Z-transform of rn cos nθ
                                                            IS
Solution.                                               M
                                                SR
                                                    z(z − cos θ)
                           w.k.t Z[cos nθ] =
                                                z2  − 2z cos θ + 1
                              Z[rn cos nθ] =    Z[cos nθ]z→ az
                                           h,
                                                                   
                                                      z(z − cos θ)
                            es
                                            =
                                                  z 2 − 2z cos θ + 1 z→ z
                                                                        a
                          ur
                                                   z z           
                                                          − cos θ
                              Z[rn cos nθ] =       r r
                        .S
                                                z2       z
                                                  2
                                                     − 2 cos θ + 1
                     r.E
                                                r        r
                                                    z(z − r cos θ)
                                            =
                                                z 2 − 2rz cos θ + r2
                  D
              h       nπ i     az
22.Show that Z an sin      = 2
                       2    z + a2
Solution.
                                        z sin θ
              w.k.t Z [sin nθ] =
                                       z2
                                     − 2z cosθ +    1
                                                 π
                    h nπ i              z sin                  z(1)          z
                                                2π 
                                                                              T
                  Z sin        =                          = 2            = 2
                           2     z 2 − 2z cos          +1  z − 2z(0) + 1  z +1
                                                                         IS
                                                  2
        By Scaling Property
                   Z[an f (n)] = F
                                    z 
                                                                    M
                                                   SR
                                      a
                                      z
                 h
                   n     nπ  i
                                      a             az
               Z a sin         =     2      = 2
                           2       z
                                   a2
                                        +1       z + a2
                                              h,
                            es
                 h
                     nnπ i     z2
23. Show that Z a cos      = 2
                       2    z + a2
                          ur
                                                      z 2 − z cos θ
                     r.E
                                Z [cos nθ] =
                                                  z 2 − 2z cos θ + 1
               D
                                                  π 
                                   2
              h    nπ i       z − z cos              z 2 − z(0)    z2
             Z cos      =               π 2    = 2            = 2
                    2     z 2 − 2z cos        +1  z − 2z(0) + 1  z +1
                                         2
                                                    2
                                                       z
                            h          nπ i            a2                   z2
                          Z an cos            =   z2
                                                                    =
                                                                        z 2 + a2
                                                       
                                       2          a2
                                                           +1
10
     Theorem
                d
     Z[nk ] = −z Z[nk−1 ] where k being any positive integer.
                dz
     Proof. By the definition of Z-transform we have
                                                            ∞
                                                            X
                                              k
                                         Z[n ] =                  nk z −n                          (1)
                                                            n=0
                                                            X∞
                                        Z[nk−1 ] =                nk−1 z −n                        (2)
                                                            n=0
                                                                                 T
                                                  k−1
                                    =         n         (−n)z          = −z
                                                                           IS
                                        n=0                                        n=0
                                       1
                                    = − Z[nk ]
                                       z                           M
                                                            SR
                      d
=⇒      Z(nk ) = −z      Z(nk−1 )
                      dz
                                                  h,
                z
24. Z[n] =
                               ur
             (z − 1)2
                             .S
                               d
             w.k.t Z[nk ] = −z    Z[nk−1 ]
                               dz
                          r.E
                                                      
                               d             d      z
                     Z[n] = −z Z(1) = −z
                               dz            dz z − 1
                   D
                                                                  
                                 (z − 1)(1) − z(1)             −1           z
                          = −z                       = −z              =
                                     (z − 1)2               (z − 1)2     (z − 1)2
                z2 + z
25. Z[n2 ] =
               (z − 1)3
                                  d
               w.k.t Z[nk ] = −z     Z[nk−1 ]
                                  dz
                                  d
                      Z[n2 ] = −z Z(n)
                                  dz 
                                                        (z − 1)2 (1) − z 2(z − 1)
                                                                                
                                  d       z
                             = −z                = −z
                                  dz (z − 1)2                    (z − 1)4
                                                                       z2 + z
                                                              
                                    z − 1 − 2z          −z − 1
                             = −z                = −z              =
                                     (z − 1)3           (z − 1)3      (z − 1)3
                                                                                      11
                         2z
26. Z[n(n − 1)] =
                      (z − 1)3
               z 3 + 4z 2 + z
27. Z[n3 ] =
                  (z − 1)4
                                         d
                   w.k.t Z[nk ] = −z        Z[nk−1 ]
                                         dz
                                         d
                           Z[n3 ] =   −z Z(n2 )
                                         dz 
                                                                   T
                                               z2 + z
                                                        
                                         d
                                                              IS
                                 =    −z
                                         dz (z − 1)3
                                                         M
                                           (z − 1)3 (2z + 1) − (z 2 + z)3(z − 1)2
                                                                                 
                                 =    −z
                                                          (z − 1)6
                                                  SR
                                           (z − 1)(2z + 1) − 3(z 2 + z)
                                                                         
                                 =    −z
                                                      (z − 1)4
                                             h,
                                          2
                                           2z − z − 1 − 3z 2 − 3z
                                                                     
                                 =    −z
                               es
                                                   (z − 1)4
                                          2
                                                               z 3 + 4z 2 + z
                                                          
                                           −z − 4z − 1
                                      −z
                             ur
                                 =                          =
                                              (z − 1)4            (z − 1)4
                           .S
                                    1
                        r.E
                                          1      A   B
                                               =   +
                                      n(n + 1)   n n+1
                                 A(n + 1) + Bn = 1
                                 n=0                 =⇒ A = 1
                                 n = −1              =⇒ B = −1
                                                            
                               1                 1          1
                          Z                = Z      −Z
                            n(n + 1)             n         n+1
                                                                      
                                                     z                z
                                           = log           − z log
                                                   z−1               z−1
                                                               
                                                            z
                                           = (1 − z) log
                                                           z−1
12
                                      1
29. Find the Z transform of                    (A.U. Nov/Dec 2006)
                                (n + 1)(n + 2)
Solution. Let
                                        1           A   B
                                                =     +
                                 (n + 1)(n + 2)   n+1 n+2
                            A(n + 2) + B(n + 1) = 1
                                n = −1              =⇒ A = 1
                                n = −2              =⇒ B = −1
                                                      
             1                         1             1
     Z                      =   Z           −Z
       (n + 1)(n + 2)                n+1            n+2
                                 ∞
                                X 1 1              ∞
                                                  X 1 1
                            =                  −
                                n=0
                                     n + 1 z n n=0 n + 2 z n
                                                                  T
                                                                                 
                                     11 1 1                  1 11 1 1
                                                             IS
                            =   1+       +       + ··· −       +     +      + ···
                                     2 z 3 z2                2 3 z 4 z2
                            =   z
                                  
                                    1 1 1
                                      +       +
                                    z 2 z2 3 z3
                                                 1 1
                                                         M   
                                                     + ··· − z
                                                                   
                                                                  2 1 1
                                                                          +
                                                                            1 1
                                                                                 +
                                                                     2 z2 3 z3 4 z4
                                                                                    1 1
                                                                                        + ···
                                                                                              
                                                 SR
                                                                          
                                               1       2               1     1
                            =   −z log 1 −         − z − log 1 −           −
                                               z                       z     z
                                           h,
                                                                  
                                               1                   1
                            =   −z log 1 −         + z 2 log 1 −       +z
                            es
                                               z                   z
                                                          
                                                    z−1
                                z + (z 2 − z) log
                          ur
                            =
                                                      z
                        .S
                     r.E
Z −1 [F (z)] = f (n)
                                                    Examples
                                                 
                       −1            10z
Example 1. Find Z
                                (z − 1)(z − 2)
                                                                           T
                                                                          IS
Solution.
                                               
                  Z   −1            10z
                               (z − 1)(z − 2)                      M
                                                      = Z −1 [F (z)]
                                                              SR
                                            10z
                                         F (z) =
                                       (z − 1)(z − 2)
                                                      h,
                               F (z)         10
                                     =
                                 z     (z − 1)(z − 2)
                              es
                            10           A       B
                                     =       +
                            ur
                                   Put z = 1              A = −10 ,
                       r.E
                                        z=2      B = 10
                                        F (z)    −10    10
                 D
                                              =      +
                                          z     z−1 z−2
                                                −10z    10z
                                        F (z) =       +
                                                z − 1 z − 2                
                                     −1              −1   z          −1    z
                                    Z [F (z)] = −10Z           + 10Z
                                                        z−1               z−2
                                     −1              n   n        n     n
                                    Z [F (z)] = 10(−1 + 2 ) = 10(2 − 1 )
                                                                 z
Example 2. Find the inverse transform of
                                                         z2   − 7z + 12
Solution.
                                             
                       −1            z
                   Z             2
                                                      = Z −1 [F (z)]
                                z − 7z + 12
                                                                 z              z
                                         F (z) =                        =
                                                         z2   − 7z + 12   (z − 4)(z − 3)
14
                                      F (z)         1
                                            =
                                        z     (z − 4)(z − 3)
                            1           A   B
                                    =     +
                     (z − 4)(z − 3)   z−4 z−3
                A(z − 3) + B(z − 4) = 1
                                 Put z = 3           B = −1 ,
                                      z=4      A=1
                                      F (z)      1      1
                                            =       −
                                        z     z−4 z−3
                                                 z      z
                                      F (z) =       −
                                              z − 4 z − 3          
                                   −1           −1    z      −1    z
                                  Z [F (z)] = Z           −Z
                                                    z−4           z−3
                                   −1          n    n
                                  Z [F (z)] = 4 − 3
                                                                     T
                                                                IS
                              z(z 2 − z + 2)
                                              
Example 3. Find Z   −1
                             (z + 1)(z − 1)2                M
                                                     SR
Solution.
                              z(z 2 − z + 2)
                                              
                                               h,
                    −1
                Z                                  = Z −1 [F (z)]
                             (z + 1)(z − 1)2
                           es
                                                  z(z 2 − z + 2)
                                         F (z) =
                                                 (z + 1)(z − 1)2
                         ur
                                         F (z)      z2 − z + 2
                                               =
                       .S
                                           z     (z + 1)(z − 1)2
                                 z2 − z + 2
                    r.E
                                                   A         B     C
                                             2
                                               =        +        +
                              (z + 1)(z − 1)     z + 1 (z − 1) (z − 1)2
              D
               Substituting z = 1                  2C = 2 =⇒ C = 1 ,
                                   z = −1          4A = 4 =⇒ A = 1
                                     z=0           A − B + C = 2,
                                                   1 − B + 1 = 2 =⇒ B = 0
                                     F (z)           1             1
                                           =              +0+
                                       z           z+1          (z − 1)2
                                                     z         z
                                     F (z) =              +
                                                   z + 1 (z − 1)2
                                                                               
                                                           z                z
                                Z −1 [F (z)] =     Z −1
                                                                −Z  −1
                                                          z+1            (z − 1)2
                                Z −1 [F (z)] =     (−1)n + n
                                                                                               15
                                 z2 − z
                                                   
                    −1
Example 4. Find Z
                             (z + 1)(z 2 + 1)
Solution.
                                              z2 − z
                                                            
                                 −1
                             Z                                   = Z −1 [F (z)]
                                          (z + 1)(z 2 + 1)
                                                                   z2 − z
                                                       F (z) =
                                                               (z + 1)(z 2 + 1)
                                                      F (z)         z−1
                                                             =
                                                        z      (z + 1)(z 2 + 1)
                                                z−1              A      Bz + C
                                                             =       +
                                           (z + 1)(z 2 + 1)    z+1       z2 + 1
Substituting z = −1 2A = −2 =⇒ A = −1 ,
                                                                             T
                               z=0                      A + C = −1
                                                                          IS
                                                        −1 + C = −1 =⇒ C = 0
                                            z=1
                                                                     M
                                                        2A + 2B + 2C = 0
                                                             SR
                                                        −2 + 2B + 0 = 0 =⇒ B = 1
                                            F (z)           1       z
                                                  =     −       + 2
                                              z           z+1 z +1
                                                     h,
                                                            z      z2
                                             F (z) =    −       + 2
                           es
                                                          z + 1 z + 1        2 
                                                                 z               z
                                  Z −1 [F (z)] =        −Z  −1
                                                                        +Z −1
                         ur
                                                                z+1             2
                                                                               z +1
                                                                      nπ
                       .S
                                                             z(z + 1)
Example 5. Find the inverse z-transform of                            .
                D
                                                             (z − 1)3
Solution.
                                           z(z + 1)
                    F (z) =
                                           (z − 1)3
                    F (z)                   (z + 1)     (z − 1 + 2)       (z − 1)      2
                          =                          =               =          3 +
                      z                    (z − 1)3       (z − 1)3       (z − 1)    (z − 1)3
                    F (z)                      1            2
                          =                        2 +
                      z                    (z − 1)      (z − 1)3
                                               z           2z
                    F (z) =                        2 +
                                           (z − 1)      (z − 1)3
                                                                            
                                                     z                   2z
               Z −1 [F (z)] =              Z −1
                                                             +Z  −1
                                                  (z − 1)3            (z − 1)3
               Z −1 [F (z)] =              n + n(n − 1) = n2
16
                                                                    T
                         Z[y(n)]      =   F (z)
                                                               IS
                     Z [y(n + 1)]     =   zF (z) − zy(0)
                     Z [y(n + 2)]     =   z 2 F (z) − z 2 y(0) − zy(1)
                     Z [y(n + 3)]     =                   M
                                          z 3 F (z) − z 3 y(0) − z 2 y(1) − zy(2)
                                                  SR
                                            Examples
                                            h,
                           es
                              Z[y(n)] = F (z)
                          Z [y(n + 1)] = zF (z) − zy(0)
                          Z [y(n + 2)] = z 2 F (z) − z 2 y(0) − zy(1)
                                       z(z − 3)                z
                      F (z) =                          =
                                 (z − 2)(z − 3)(z − 4)   (z − 2)(z − 4)
                      F (z)         1             A       B
                            =                =        +
                        z     (z − 2)(z − 4)   (z − 2) (z − 4)
⇒ A(z − 4) + B(z − 2) = 1
                                                                             1
                  Put z = 2       ⇒          −2A = 1      ⇒         A=−
                                                                             2
                                                                             1
                  Put z = 4,      ⇒           2B = 1      ⇒             B=
                                                                             2
                                                              T
                          F (z)     1 A          1 B
                                = −
                                                          IS
                                              +
                            z       2 (z − 2) 2 (z − 4)
                                                       M
                                                       
                                    1     z       1     z
                          F (z) = −             +
                                    2 z−2         2 z−4
                                               SR
                                                              
                       −1           1 −1      z       1 −1    z
                      Z [F (z)] = − Z               + Z
                                    2       z−2       2      z−4
                                         h,
                                    1        1
                          y(n) = − (2)n + (4)n
                                    2        2
                           es
                         ur
                              Z[y(n)] = F (z)
                          Z [y(n + 1)] = zF (z) − zy(0)
                          Z [y(n + 2)] = z 2 F (z) − z 2 y(0) − zy(1)
                                                      z
                   F (z) = Z [y(n)] =
                                          (z −   2)(z 2
                                                 + 6z + 9)
                              F (z)            1                    1
                                    =          2
                                                           =
                                z     (z − 2)(z + 6z + 9)    (z − 2)(z + 3)2
                              F (z)     A        B        C
                                    =       +       +
                                z     z − 2 z + 3 (z + 3)2
                                                                             1
                   Put z = −3         ⇒          −5C = 1    ⇒          C=−
                                                                             5
                                                                            1
                   Put z = 2,         ⇒           25A = 1   ⇒          A=
                                                                            25
                                                                T
                                                                  1
                                                            IS
                             A + B = 0 ⇒ B = −A ⇒ B = −
                                                                  25
                                                            M
                                                    SR
                                                                     
                    F (z)    1     1          1      1       1      1
                          =               −               −
                      z     25 z − 2         25 z + 3        5 (z + 3)2
                                                                     
                             1     z          1      z       1      z
                                            h,
                    F (z) =               −               −
                            25 z − 2         25 z + 3        5 (z + 3)2
                              es
                                                                            
                 −1          1 −1       z          1 −1      z        1     z
                Z [F (z)] =    Z               − Z                 −
                            25        z−2         25       z+3        5 (z + 3)2
                            ur
                            (2)n (−3)n 1 1
                                                                      
                                                               (−3)z
                          .S
                                                         −1
                          =      −           −         Z
                             25        25       5 (−3)        (z + 3)2
                       r.E
                                               
                                          az
                            ∵Z   −1
                                                  = n(a)n
                                      (z − a)2
                   D
                                 1         1      1
                             =      (2)n − (−3)n + n(−3)n
                                 25       25      15
                                Z[y(n)] = F (z)
                            Z [y(n + 1)] = zF (z) − zy(0)
                                                                                             19
                             z (z − 2)
               F (z) =
                       (z − 3)(z + 1)(z + 3)
                                                                T
               F (z)          (z − 2)             A       B       C
                     =                       =        +       +
                                                          IS
                 z     (z − 3)(z + 1)(z + 3)   (z − 3) (z + 1) (z + 3)
                                                       M
               ⇒ A(z + 3)(z − 3) + B(z + 1)(z − 3) + C(z + 1)(z + 3) = z − 2
                                                SR
                                                                             3
                Put z = −1         ⇒       −8A = −3         ⇒           A=
                                                                             8
                                         h,
                                                                             5
                            es
                                                                             1
                Put z = 3,         ⇒         24C = 1        ⇒           C=
                        .S
                                                                             24
                     r.E
                                                                
                   F (z)    3    1         5     1       1      1
                ∴         =            −              +
                     z      8 z+1         12 z + 3      24 z − 3
                 D
                                                                           
               −1           3 −1     z         5 −1     z          1 −1     z
             Z [F (z)] =      Z             − Z                + Z
                            8       z+1       12       z+3        24       z−3
                                                                          
               −1            1 −1      z        1 −1      z         1     z
             Z [F (z)] =       Z             − Z                −
                            25       z−2        25      z+3         5 (z + 3)2
                            3           5          1
             Z −1 [F (z)] =   (−1)n − (−3)n + (3)n
                            8          12          24
                                      Z[y(n)] = F (z)
                                  Z [y(n + 1)] = zF (z) − zy(0)
                                  Z [y(n + 2)] = z 2 F (z) − z 2 y(0) − zy(1)
                                            z (z − 4)
                                F (z) =
                                             (z − 2)2
                                                                          T
                                F (z)        (z − 4)     z−2−2
                                                                     IS
                                      =              2 =
                                  z         (z − 2)        (z − 2)2
                                F (z)
                                      =
                                             (z − 2)
                                                     2 −
                                                              2 M2 =
                                                                         1
                                                                             −
                                                                                2
                                                        SR
                                  z         (z − 2)      (z − 2)       z − 2 (z − 2)2
                                                                 
                                                 z            2z
                                F (z) =                −
                                              z−2          (z − 2)2
                                                  h,
                                                                           
                                                      z                 2z
                          Z −1 [F (z)] =    Z −1
                                                            −Z  −1
                                   es
                                                    z−2              (z − 2)2
                                                n        n       n
                                y(n) =      (2) − n(2) = (2) (1 − n)
                                 ur
                               .S
Example 5. Solve the difference equation y(n + 3) − 3y(n + 1) + 2y(n) = 0 given that
                            r.E
                                 Z[y(n)]    =   F (z)
                             Z [y(n + 1)]   =   zF (z) − zy(0)
                             Z [y(n + 2)]   =   z 2 F (z) − z 2 y(0) − zy(1)
                             Z [y(n + 3)]   =   z 3 F (z) − z 3 y(0) − z 2 y(1) − zy(2)
                                       4z (z 2 − 1)
                  F (z) = Z [y(n)] =
                                      (z 3 − 3z + 2)
                              F (z)         4(z 2 − 1)      4(z + 1)(z − 1)
                                    = =          2        =
                                z        (z − 1) (z + 2)    (z − 1)2 (z + 2)
                              F (z)       4(z + 1)          A          B
                                    =                  =        +
                                z     (z − 1)(z + 2)     (z − 1) (z + 2)
                              ⇒ 4(z + 1) = A(z + 2) + B(z − 1)
                                                                             8
                  Put z = 1          ⇒          3A = 8         ⇒        A=
                                                                             3
                                                                             4
                                                                T
                  Put z = −2,        ⇒        −3B = −4         ⇒        B=
                                                                             3
                                                           IS
                           F (z)
                                 =
                                   8
                                     
                                         1
                                            
                                               +
                                                 4
                                                   
                                                       1 M
                                                         
                                                SR
                             z     3 z−1         3 z+2
                                                      
                                   8     z       4     z
                           F (z) =             +
                                   3 z−1         3 z+2
                                           h,
                                                             
                        −1         8 −1      z       4 −1    z
                       Z [F (z)] =   Z             + Z
                            es
                                   3       z−1       3      z+2
                                   8 n 4
                                     (1) + (−2)n
                          ur
                           y(n) =
                                   3        3
                        .S
Example 6.Solve the difference equation y(n) + 3y(n − 1) − 4y(n − 2) = 0, n > 2 given that
y(0) = 3, y(1) = −2, by the method of Z - transform.
                     r.E
Replace n by n + 2, we get
                              Z[y(n)] = F (z)
                          Z [y(n + 1)] = zF (z) − zy(0)
                          Z [y(n + 2)] = z 2 F (z) − z 2 y(0) − zy(1)
                                   z (3z + 7)
                          F (z) =
                                  z 2 + 3z − 4
                          F (z)       (3z + 7)        A       B
                                =                =        +
                            z     (z + 4)(z − 1)   (z + 4) (z − 1)
⇒ 3z + 7 = A(z − 1) + B(z + 4)
                   Put z = 1          ⇒          5B = 10          ⇒        B=2
                   Put z = −4,        ⇒         −5A = −5          ⇒        A=1
                                                                  T
                                                         
                              F (z)         1           1
                                                              IS
                                    = 1          +2
                                z         z+4          z−1
                              F (z) = 1
                                        
                                            z
                                               
                                                 +2
                                                     
                                                        zM  
                                                  SR
                                          z+4          z−1
                                                               
                           −1           −1    z          −1    z
                          Z [F (z)] = Z            + 2Z
                                             z+4              z−1
                                             h,
                                           n       n
                              y(n) = (−4) + 2(1)
                             es
                           ur
where f (z) = z n−1 F (z) and C is the contour which include all the poles of f (z) .
                  D
                                      1     dm−1 
                                                   (z − a)m z n−1 F (z)
                                                                       
                       R(a) = lim              m−1
                               z→a (m − 1)! dz
                                             Examples
                                                      10z
Example 1. Find the inverse z-transform of                      by residue method.
                                                 (z − 1)(z − 2)
                                                                                       23
                           10z
Solution. Let F (z) =
                     (z − 1)(z − 2)
 −1
Z [F (z)] = sum of the residues , z = 1, 2 are the simple poles (or) pole of order 1
R(1) = −10(1n )
                                                                 T
                                                                       
                                                               10z
                                                            IS
                                                   n−1
                                    = lim (z − 2)z
                                      z→2                (z − 1)(z − 2)
                                    = lim (z − 2)
                                                  
                                                        M10z n
                                                    (z − 1)(z − 2)
                                                                   
                                                 SR
                                      z→1
                                                 n
                                                   
                                             10z
                               R(1) = lim             = 10(2)n
                                      z→2 (z − 1)
                                           h,
                                                z(z + 1)
                        .S
                        z(z + 1)
Solution. Let F (z) =
                        (z − 1)3
                 D
                                        1 d2               n−1 2
                                                                       
                                                        3z     (z + z)
                        R(1) =     lim           (z − 1)
                                   z→1 2! dz 2               (z − 1)3
                                        1 d2  n+1
                                                     + zn
                                                           
                              =    lim       2
                                                z
                                   z→1 2! dz
                                         1 d 
                                               (n + 1)nz n + nz n−1
                                                                      
                              =    lim
                                   z→1 2! dz
                                         1 
                                            (n + 1)nz n−1 + n(n − 1)z n−2
                                                                          
                              =    lim
                                   z→1 2!
                                    1
                              =       [(n + 1)n + n(n − 1)]
                                   2!
                                    1 2
                              =       (n + n + n2 − n) = n2
                                   2!
24
Convolution. Let {f (n)} and {g(n)} be any two sequences. Then the convolution of these
sequence is defined as another sequence given by
                                                        n
                                                        X
                                    f (n) ∗ g(n) =            f (r)g(n − r)
                                                        r=0
                                                                         T
           F (z) G(z) = Z[f (n)] Z[g(n)]
                                                                    IS
                        "∞             #" ∞          #
                         X               X
                      =      f (n)z −n      g(n)z −n
                              n=0                 n=0           M
                                                        SR
                            ∞
                            X
                       =          [f (0)g(n) + f (1)g(n − 1) + f (2)g(n − 2) + · · · + f (n)g(0)] z −n
                            n=0
                                                h,
                             ∞
                                " n                     #
                            X    X
                       =                  f (r)g(n − r) z −n
                             es
                            n=0     r=0
                             n
                           ur
                            X
     w.k.t f (n) ∗ g(n) =         f (r)g(n − r)
                         .S
                            r=0
                            ∞
                            X
                                  [f (n) ∗ g(n)] z −n = Z [f (n) ∗ g(n)]
                      r.E
                       =
                            n=0
                  D
Formula
                                                                    1 − an+1
                              1 + a + a2 + a3 + · · · + an =
                                                                      1−a
                                                                                                                          25
                                  z2
                                                                                     
                   −1                                      −1          z       z
               Z                                     = Z
                            (z − a)(z − b)                          (z − a) (z − b)
           By Convolution Theorem
                                                                                           
                                                           −1          z         −1      z
                                                     = Z                      ∗Z
                                                                    (z − a)           (z − b)
                                                     = an ∗ b n
              By Defn. Convolution
                                                                                       T
                                                         n
                                                                                IS
                                                         X
                                 f (n) ∗ g(n) =                f (r)g(n − r)
                                                         r=0
                                                          n             M       n                  n              a r
                                                               SR
                                                         X                      X                  X
                                        n        n              r n−r                 r n −r                 n
                                       a ∗b          =         ab        =            ab b     =         b
                                                         r=0                    r=0
                                                                                           b       r=0
                                                              a   a 2          a n 
                                                     = bn 1 +      +       + ··· +
                                                         h,
                                                                b     b              b
                               es
                                                                 bn+1 − an+1
                                                                     a n+1 
                                                                                       
                             ur
                                            1−
                                     = bn        b         n    bn+1
                                                    a =b 
                                                                            
                           .S
                                                                    b−a
                                                                            
                                              1−
                                                    b                 b
                        r.E
                                              n+1      n+1
                                                            
                                              b    −  a
                                     = bn+1 n
                                              b b (b − a)
                D
                            2            n+1
                                             − an+1
                                  
               −1         z            b
             Z                       =
                    (z − a)(z − b)         b−a
                                                                                      z2
                                                                                                  
                                                                       −1
Example 2. Using convolution theorem evaluate Z
                                                                                (z − 1)(z − 3)
Solution.
   Derive the solution of Example 1
                                                      z2           bn+1 − an+1
                                                              
                                       −1
                                   Z                             =
                                                (z − a)(z − b)        b−a
   Put a = 1, b = 3 we get
                                                                                              8z 2
                                                                                                          
                                                                            −1
Example 3. Using convolution theorem evaluate Z
                                                                                        (2z − 1)(4z − 1)
Solution.
                                                                                                          
                                                8z 2                                  z2
                                                                
                                 −1
                                                                      = Z −1 
                                                                                              
                             Z                                                             
                                          (2z − 1)(4z − 1)                        1        1 
                                                                                z−       z−
                                                                                   2        4
     Derive the solution of Example 1
                                                                 z2           bn+1 − an+1
                                                                         
                                                  −1
                                              Z                             =
                                                           (z − a)(z − b)        b−a
                                                                                             T
             1    1
     Put a = , b = we get
                                                                                        IS
             2    4
                                                                n+1  n+1          n+1  n+1
            
                         z2
                                                   
                                                                1
                                                                4
                                                                      −
                                                                           1
                                                                           2
                                                                                M      1
                                                                                       4
                                                                                             −
                                                                                                  1
                                                                                                  2
                                                                       SR
            −1
                                  
        Z                      =                                         =          
                     1        1                                  1       1                   1
                   z−       z−                                        −                    −
                      2        4                                   4       2                   4
                                                                 h,
                                                                                     "           2(n+1) #
                                                                                           n+1
                                                                n+1  n+1
                                                                1          1             1          1
                                                                      −                        −
                                        es
                                                                2          4             2          2
                                                           =          2          =           2
                                                                      1                         1
                                      ur
                                                                      2                         2
                                    .S
                                                               "                       #
                                                                    n+1−2       2n+2−2
                                                                 1             1
                                                           =              −
                                 r.E
                                                                 2             2
                                                               "         2n #
                                                                    n−1
                                8z 2
                                                  
                                                                 1          1
                           D
                 −1
             Z                                             =            −
                          (2z − 1)(4z − 1)                       2          2
                                                                                              8z 2
                                                                                                          
                                                                            −1
Example 4. Using convolution theorem evaluate Z
                                                                                        (2z − 1)(4z + 1)
Solution.
                                                 8z 2                              z2
                                                                                            
                                  −1                                       −1
                              Z                                       = Z
                                           (2z − 1)(4z + 1)                 (z − 12 )(z + 14 )
                                                                           n "       n+1 #
                                                                        2 1              1
                                                                      =         1− −
                                                                        3 2              2
                                                                 z2           bn+1 − an+1
                                                                         
                                                  −1
                                              Z                             =
                                                           (z − a)(z − b)        b−a
                                                                                                        27
              1        1
   Put a = , b = − , we get
              2        4
                                 n+1  n+1                n+1  n+1
                                    1            1              1           1
                           
                  2               −         −                 −         −
                z                   4            2              4           2
Z −1 
                           
                         =                      =               
            1          1              1        1                      3
        z−          z+                −     −                       −
             2          4               4        2                      4
                                  "                       #
                                          n+1    n+1                n    n 
                                4     1              1          4     1     1           1   1
                              =               − −             =                  − −      −
                                3     2              4          3     2     2           4   4
                                         n                 n             n            n
                 8z 2
                                                                       
     −1                         2     1         1       1         2 1        2    1
   Z                          =            +          −        =           +    −
          (2z − 1)(4z + 1)      3     2         2       4         3 2        3    4
                                                                                                
                                                                                   1         
Example 7. Using convolution theorem and evaluate Z −1 
                                                                                −1
                                                                                           
                                                                                           −1 
                                                                               z         z
                                                                            T
                                                                            1−        1+
                                                                                 2         4
                                                                     IS
Solution.
                                                                 M
                                                               SR
                                                                          
                   1                              1
Z −1                          = Z −1  
                                                         
                                                            
             z −1       z −1 
                                                   
                                                   h,
                                              1        1 
          1−         1+                    1−       1+
               2          4                   2z       4z
                              es
                                                           
                                                                                                 2
                            ur
                                                                                                        
                                                 −1
                                                               1                −1          8z
                                           = Z         2z − 1   4z + 1   = Z
                                                                          
                                                                                       (2z − 1) (4z + 1)
                          .S
                                                          2z         4z
                       r.E
                         z2
                              
                       −1
Example 9. Find Z                .
                      (z − a)2
                                  
                         −1     z
Solution. We know that Z             = an
                              z−a
                                   z2
                                                                     
                       −1                             −1   z       z
                   Z                           = Z
                                (z − a)2                (z − a) (z − a)
                                                                               
                                                   −1      z         −1      z
                                               = Z                ∗Z
                                                        (z − a)           (z − a)
                                                             n
                                                             X             Xn
                                               = an ∗ an =      ar an−r =      ar an a−r
                                                               r=0          r=0
                                               = an + an + · · · + an    (n + 1) times
                                               = (n + 1)an
28
                          z2
                                       
                                   −1
Example 10. Find Z                .
                       (z + a)2
                                   
                         −1    z
Solution. We know that Z              = (−a)n
                              z+a
                                  z2
                                                                         
                      −1                                    −1 z       z
                 Z                                 = Z
                               (z + a)2                     (z + a) (z + a)
                                                                                   
                                                       −1      z         −1      z
                                                   = Z                ∗Z
                                                            (z + a)           (z + a)
                                                                        Xn
                                                   = (−a)n ∗ (−a)n =        (−a)r (−a)n−r
                                                                                     r=0
                                                   = (−a) + (−a) + · · · + (−a)n
                                                                 n             n
                                                                                                (n + 1) times
                                                   = (n + 1)(−a)n
                           z2
                                       
                                   −1
Example 11. Find Z                .
                                                                                           T
                       (z − 1)3
                                                                                           IS
                                   
                         −1    z
Solution. We rnow that Z              = 1n
                              z−1
                                                                                     M
                                                                           SR
                                               z2
                                                                                          
                                   −1                                 −1       z       z
                               Z                            =        Z
                                            (z − 1)3                        (z − 1) (z − 1)2
                                                                                                    
                                                                               z                 z
                                                                     h,
                                                                       −1                −1
                                                            =        Z                ∗Z
                                                                            (z − 1)           (z − 1)2
                                      es
                                                            =        1n ∗ n = n ∗ 1n
                                                                      n
                                    ur
                                                                     X
                                                            =            r1n−r = 1 + 2 + · · · + n
                                  .S
                                                                     r=0
                                                                     n(n + 1)
                                                            =
                               r.E
                                                                        2
                       D
                             z3                                         z3
                                                                           
                                   −1                           −1
Example 12. Find Z                      and hence deduce Z                      .
                          (z − a)3                                   (z − 1)3
                                                           z2
                                                                
                           −1      z          n    −1
Solution. We know that Z                   = a ,Z                    = (n + 1)an
                                z−a                     (z − a)2
                  z3                       z2
                                                      
          −1                    −1                  z
        Z                 = Z
               (z − a)3                (z − a)2 (z − a)
                                           z2
                                                                  
                                −1                    −1       z
                          = Z                     ∗Z
                                       (z − a)2            (z − a)
                                          n   n
                          = (n + 1)a ∗ a
                              Xn                       X n                          n
                                                                                    X
                                     r          n−r          r          n −r      n
                          =       a (r + 1)a         =      a (r + 1)a a = a          (r + 1)
                                                  r=0                              r=0                     r=0
                                                   n
                                            = a [1 + 2 + · · · + (n + 1)]
                           3
                                  
                        z                        (n + 1)(n + 2)
          Z −1                              = an
                     (z − a)3                          2
                                                                                                         29
                          z3
                                 
              −1                                   (n + 1)(n + 2)
          Z                                 = 1n
                       (z − 1)3                          2
                                    z3
                                              
                                −1
Example 13. Find Z                                using Convolution Theorem.
                            (z − 2)2 (z − 3)
                                                                z2
                                                                    
                             −1      z           n    −1
Solution. We know that Z                    = a ,Z                       = (n + 1) an
                                   z−a                       (z − a)2
              z3                          z2
                                                          
   −1                             −1                   z
 Z                         = Z                   .
        (z − 2)2 (z − 3)               (z − 2)2 (z − 3)
                                          z2
                                                                    
                                  −1                     −1       z
                           = Z                      ∗Z
                                       (z − 2)2                (z − 3)
                                          n     n
                           = (n + 1) 2 ∗ 3
                                n                             n                            n           r
                               X
                                               r n−r
                                                            X
                                                                          r n −r        n
                                                                                          X            2
                           =        (r + 1) 2 .3        =        (r + 1) 2 .3 .3 = 3          (r + 1)
                               r=0                          r=0                           r=0
                                                                                                       3
                                                                         T
                                   "                   2           3                     n  #
                                n             2            2           2                        2
                           = 3 1+2                 +3            +4         + · · · + (n + 1)
                                                                         IS
                                              3            3           3                        3
Let x =
          
           2                                                             M
                                                                 SR
           3
                   S − xS = 1 + x + x2 + x3 + · · · + xn − (n + 1)xn+1
                                                                    
                                      es
                                   1 − xn+1
                                             
                 (1 − x) S =                     − (n + 1)xn+1
                                    ur
                                     1−x
                                  .S
                                (1 − xn+1 ) (n + 1)xn+1
                         S =                  −
                                  (1 − x)2           (1 − x)
                               r.E
                                               
                                            n+1
                                  1 − 23
                                                                   n+1
                                                       (n + 1) 23
                           =                2 −
                         D
                                                             1 − 23
                                                                    
                                     1 − 23
                                  "       n+1 #                      n+1
                                            2                          2
                           = 9 1−                      − 3(n + 1)
                                            3                          3
                                        n+1                     n+1
                                         2                          2
                           = 9−9                   − 3(n + 1)
                                         3                          3
                                                       n+1
                                                         2
                           = 9 − (9 + 3n + 3)
                                                         3
                                                   n+1
                                                     2
                           = 9 − (3n + 12)
                                                     3
                                                  n+1
                                                    2
                         S = 9 − 3 (n + 4)
                                                    3
                                              z3
                                                          
                               −1
                           Z                                   = 3n .S
                                        (z − 2)2 (z − 3)
30
                                                            "              n+1 #
                                                                             2
                                                      = 3n 9 − 3 (n + 4)
                                                                             3
                                                                              n+1
                                                          n+2    n+1           2
                                                      = 3     −3     (n + 4)
                                                                               3
                                                          n+2            n+1
                                                      = 3     − (n + 4) 2
                                                                             T
                                  ∞
                                                                      IS
                                  X
                                =     f (n)z −n
                                         n=0
                                                                  M
                                     = f (0) + f (1)z −1 + f (2)z −2 + · · · +
                                                         SR
                                               f (1) f (2)
                                     = f (0) +       + 2 + ···
                                                 z       z
                                                    h,
As z → ∞ we get
                            es
                                                                                    
                                                 f (1) f (2)
                         lim F (z) = lim f (0) +      + 2 + ···
                        z→∞          z→∞           z    z
                          ur
                                   = f (0)
                        .S
n→∞ z→1
                                                ∞
                                                X
                         Z[f (n + 1) − f (n)] =   [f (n + 1) − f (n)]z −n
                                                           n=0
                                                ∞
                                                X
                      Z[f (n + 1)] − Z[f (n)] =   [f (n + 1) − f (n)]z −n
                                                           n=0
                                                           X∞
                      z[F (z) − f (0)] − F (z) =                 [f (n + 1) − f (n)]z −n
                                                           n=0
                                                ∞
                                                X
                        (z − 1)F (z) − zf (0) =   [f (n + 1) − f (n)]z −n
                                                           n=0
                                                                                                       31
As z → 1 we get
                                   ∞
                                   X
   lim (z − 1)F (z) − 1f (0) = lim   [f (n + 1) − f (n)]z −n
   z→1                           z→1
                                       n=0
                                 ∞
                                 X
                            =          [f (n + 1) − f (n)]
                                 n=0
                                     n
                                     X
                            =    lim   [f (n + 1) − f (n)]
                                 n→∞
                                        k=0
                            =    lim [[f (1) − f (0)] + [f (2) − f (1)] + · · · [f (n + 1) − f (n)]]
                                 n→∞
                            =    lim [f (n + 1) − f (0)]
                                 n→∞
    lim (z − 1)F (z) − f (0) =   lim [f (n + 1)] − f (0)
    z→1                          n→∞
           lim (z − 1)F (z) =    lim f (n + 1)
                                                                  T
           z→1                   n→∞
                                                             IS
Since lim f (n + 1) = lim f (n) we have
     n→∞             n→∞
                                                         M
                                 lim f (n) = lim(z − 1)F (z)
                                                  SR
                                 n→∞               z→1
                                              h,
                           es
                         ur
                       .S
                    r.E
                  D